Haben eine Aktion und ihre Euler-Lagrange-Gleichungen die gleichen Symmetrien?

Gehen Sie von einer bestimmten Aktion aus S mit bestimmten Symmetrien, von denen nach dem Lagrange-Formalismus die Bewegungsgleichungen (EOM) des Systems die entsprechenden Euler-Lagrange-Gleichungen sind.

Kann es sein, dass die durch dieses Verfahren abgeleiteten Bewegungsgleichungen andere Arten und/oder Anzahlen von Symmetrien haben als die Aktion, von der man ausgegangen ist? Und wenn ja, gibt es zugrunde liegende Prinzipien, warum welche Art von Symmetrien, die die Wirkung nicht hat, in entsprechenden EOMs auftreten können oder welche Art von Symmetrien der Wirkung in den EOMs, die aus den Euler-Lagrange-Gleichungen abgeleitet werden, möglicherweise verschwinden können?

Antworten (1)

Einstellung. Wir denken über eine Umgestaltung nach 1 die auf die Feldvariablen wirkt ϕ a ( X ) und möglicherweise der Raumzeitpunkt X μ . Die Transformation wiederum gelten für

  1. Die Aktion S v [ ϕ ] = v D N X   L .

  2. Die Euler-Lagrange-Gleichungen = Bewegungsgleichungen (EOM).

  3. Eine Lösung ϕ von EOM.

Definition. Wenn eines der Items 1-3 unter der Transformation invariant ist, sprechen wir von einer Symmetrie des entsprechenden Items 1-3 .

Definition. Wenn eine Lösung (3) keine Symmetrie hat, die die EOM (2) haben, sprechen wir von einer spontan gebrochenen Symmetrie .

Definition. Erinnern wir uns als nächstes an die Definition eines (off-shell 2 ) Quasi-Symmetrie der Wirkung. Das bedeutet, dass sich die Wirkung um ein Grenzintegral ändert

(0,1) S v ' [ ϕ ' ] + v ' D N 1 X   ( )   =   S v [ ϕ ] + v D N 1 X   ( )
unter der Verwandlung.

Vorschlag. Wenn eine Wirkung (1) eine Quasi-Symmetrie hat, dann muss im Allgemeinen auch die EOM (2) eine Symmetrie haben (bzgl. derselben Transformation), vgl. zB dieser Phys.SE Beitrag.

Beispiele:

  1. Ein Beispiel ist die Maxwell-Lagrange-Dichte (im Vakuum ohne die J μ A μ Quellbegriff)

    (1.1) L   =   1 4 F μ v F μ v   =   1 2 ( E 2 B 2 ) ,
    die keine elektromagnetischen hat S Ö ( 2 , R ) Dualitätssymmetrie
    (1.2) ( E , B ) ( E cos θ B Sünde θ , B cos θ + E Sünde θ ) ,
    während die Euler-Lagrange-Gleichungen (die Maxwell-Gleichungen im Vakuum) unter elektromagnetischer Dualität symmetrisch sind.

  2. Ein weiteres Beispiel ist ein nicht-relativistisches Teilchen mit freien Punkten, bei dem die Lagrange-Funktion

    (2.1) L   =   1 2 M Q ˙ 2
    ist unter der Galilei-Symmetrie nicht invariant
    (2.2) Q ˙ Q ˙ + v ,
    noch die Dilatations-/Skalensymmetrie
    (2.3) Q λ Q ,
    aber die EOM
    (2.4) Q ¨   =   0
    ist unveränderlich. Bei der Galilei-Symmetrie (2.2) ändert sich die Lagrange-Funktion um eine totale Zeitableitung
    (2.5) L L + M v D D T ( Q + v T 2 ) .
    Siehe auch diesen Phys.SE-Beitrag. Damit ist (2.2) tatsächlich ein Beispiel für eine Quasi-Symmetrie der Wirkung. [Es ist eine lehrreiche Übung, die entsprechende Noether-Ladung herzuleiten Q . Auf der infinitesimalen Ebene lautet die Galileische Transformation (2.2).
    (2.6) δ Q ˙   =   δ v   =   ε , δ Q   =   ε T , δ L   =   ε D F D T , F   :=   M Q .
    Die bloße Noether-Ladung ist
    (2.7) Q 0   =   T L Q ˙   =   T M Q ˙ ,
    dabei ist die volle Noetherladung
    (2.8) Q   =   Q 0 F   =   M ( Q ˙ T Q ) ,
    die auf der Schale konserviert wird, vgl. Satz von Noether . Der (nicht-relativistische) Galileische Boost-Generator (2.8) sollte mit den (relativistischen) Lorentz-Boost-Generatoren verglichen werden T P X E in relativistischen Theorien, vgl. zB dieser Phys.SE Beitrag.]

  3. Die Dilatations-/Maßstabstransformation

    (3.1) Q λ Q ,
    ist keine Quasi-Symmetrie der Lagrange-Wirkung
    (3.2) S [ Q ]   =   D T   L , L   =   M 2 Q ˙ 2 k 2 Q 2 ,
    für den einfachen harmonischen Oszillator (SHO), aber es ist eine Symmetrie des EOM
    (3.3) M Q ¨   =   k Q .

  4. Die Dilatations-/Maßstabstransformation

    (4.1) Q λ Q , P λ P ,
    ist keine Quasisymmetrie der Hamiltonschen Wirkung
    (4.2) S H [ Q , P ]   =   D T   L H , L H   =   P Q ˙ H , H   =   P 2 2 M + k 2 Q 2 ,
    für die SHO, aber es ist eine Symmetrie von Hamiltons EOM
    (4.3) P   =   M Q ˙ , P ˙   =   k Q .

  5. Der EOM der SHO

    (5.1) M Q ¨   =   k Q
    ist unter der zeitlichen Symmetrie nicht invariant
    (5.2) T λ T , λ     ± 1 ,
    aber die triviale Lösung Q = 0 Ist.

--

1 Beachten Sie, dass die Transformation im Hauptteil dieser Antwort nur auf die Feldvariablen wirkt ϕ a ( X ) und möglicherweise der Raumzeitpunkt X μ , was die Art der Transformation ist, die für den Satz von Noether relevant ist . An eine Transformation anderer Objekte (zB Parameter) denken wir per se nicht.

Beispiel für letzteres: Eine Transformation der Lagrange-Dichte

(6.1) L λ L , λ     1 ,
ist keine Quasi-Symmetrie der Lagrange-Dichte, aber es ist eine Symmetrie der EL-Gleichungen.

2 Hier weist das Wort Off-Shell darauf hin, dass nicht davon ausgegangen wird, dass die EOM unter der spezifischen Transformation gelten. Wenn wir im Falle kontinuierlicher Transformationen annehmen, dass der EOM gilt, dann ist jede infinitesimale Variation der Aktion trivialerweise ein Grenzintegral.

Vielen Dank an Qmechanic für diese schönen Beispiele, die beide Fälle veranschaulichen. Kennen Sie auch die Antwort auf meine Frage zur Skaleninvarianz? Und allgemeiner gesagt, gibt es zugrunde liegende Prinzipien, die besagen, warum welche Art von Symmetrien, die die Lagrange-Funktion nicht hat, in entsprechenden EOMs auftreten können oder welche Art von Symmetrien der Lagrange-Funktion möglicherweise in den aus den Euler-Lagrange-Gleichungen abgeleiteten EOMs verschwinden können?
@Qmechanic Interessante Reihe von Beispielen. In Bezug auf das erste Beispiel glaube ich nicht, dass es eine Möglichkeit gibt, die SO (2) -Transformation in Bezug auf das Vektorpotential ( A μ ). Begründung Es ist relevant, weil Sie die Aktion in Bezug auf die E- und B-Felder nicht variieren können, zumindest soweit ich weiß.
Ein weiteres Beispiel für die Transformation der Dualität ist ( M , k ) ( 1 k , 1 M ) im Falle eines linearen harmonischen 1D-Oszillators, dessen Lagrange L = 1 2 M X ˙ 2 1 2 k X 2 unter der gegebenen Transformation nicht invariant ist, sondern die entsprechende Bewegungsgleichung M X ¨ = k X Ist.
Hallo @Manas Dogra. Danke für die Rückmeldung. Vergleiche mit Gl. (6.1).